库仑势的fourier transform

问题

三维库仑势的形式为:

按动量Fourier展开:

求 $V(vec{k})$ 。

求解

但 $e^{-ikr}$ 与 $e^{-ikr}$ 在 $+infty$ 处发散。

使其收敛

为了解决发散问题,可在一开始令

所以

函数满足一致收敛,积分与极限才可交换顺序。